Solve the following equationX-3(x+2)=4 X - 3 ( x + 2 ) = 4

Answers

Answer 1

Given: X-3(x+2)=4X-3(x+2)=4

Find: the solution of the given equation

Explanation:

[tex]\begin{gathered} X-3(x+2)=4 \\ X-3x-6=4 \\ X-3x=10......................(1) \\ 4X-3(x+2)=4 \\ 4X-3x-6=4 \\ 4X-3x=10...........................(2)\text{ } \end{gathered}[/tex]

on solving equation (1) and (2) , we get

Answer 2

Answer:

x = -5

Step-by-step explanation:

Solve for 'x':

    x - 3(x + 2)= 4

    x - 3x - 3*2 = 4     {Distributive property}

   x - 3x  - 6    = 4

Combine like terms in LHS,

         -2x - 6  = 4

Add 6 to both sides,

               -2x = 4 + 6

               -2x = 10

Divide both sides by (-2)

                   x = 10 ÷ (-2)

                   [tex]\sf \boxed{x= -5}[/tex]

     


Related Questions

can you explain to me what a voronoi diagram is?and what are the edges, and other things?

Answers

A Voronoi diagram is a division of a plane into areas near each of a given set of objects in mathematics. These things are just infinitely many points in the plane in the simplest scenario (called seeds, sites, or generators). There is a region, known as a Voronoi cell, for each seed that is made up of all points in the plane that are closer to that seed than to any other point. A set of points' Voronoi diagram and Delaunay triangulation are identical.

We are aware that any intersection of half-planes results in a convex region bordered by a collection of linked line segments. Voronoi edges are the line segments that define the limits of Voronoi regions. These edges' termini are known as Voronoi vertices.

Voronoi diagram:

A Voronoi diagram in mathematics is a division of a plane into areas near to each of a given set of objects. These things are simply infinitely many plane points in the most basic scenario (called seeds, sites, or generators).

To learn more about Voronoi diagram visit: https://brainly.com/question/26309607

#SPJ9

The table lists the locations of four underwater camreas relative to sea level which camrea is the farthest from the surface of the water

Answers

The fish camera will farthest from from the surface of water as it is -15 1/3 feet away that is 15 1/3 feet below the sea level. so correct option is B.

What is number line?

A number line is a diagram of a graduated straight line used to represent real numbers in elementary mathematics. It is assumed that every point on a number line corresponds to a real number, and that every real number corresponds to a point. A number line is a visual representation of numbers on a straight line in mathematics. A number line's numbers are arranged in a sequential manner at equal intervals along its length. It is typically represented horizontally and can extend indefinitely in any direction. A number line is exactly what it sounds like—a horizontal, straight line with numbers spaced evenly apart along its length.

To know more about number line,

https://brainly.com/question/13425491?referrer=searchResults

#SPJ1

(Write an algebraic expression to model the given context. Give your answer in simplest form.)

Answers

Answer: 19t + 100

Explanation:

From the information given,

she earns $19 per hour

Last week, she worked t hours. This means that the amount earned in t hours is

t * 19 = 19t

If her friend paid her $100 for photoshoot, the expression to represent the total amount April earned last week is

19t + 100

Rewrite in simplest terms: 9(-7t + 4t — 10) — 9t

Answers

The simplest form of the given equation [9(-7t + 4t — 10) — 9t] is [-36t - 90].

What do we mean by simplest terms?

If a fraction's top and bottom have only one other common factor, it is said to be in its simplest form.

In other words, the top and bottom cannot be divided further and remain whole numbers.

The simplest form is sometimes referred to as "lowest terms."

So, the simplest form of the equation:

9(-7t + 4t — 10) — 9t

Now, calculate to simplest terms as follows:

9(-7t + 4t — 10) — 9t

-63t + 36t - 90 - 9t

-63t + 27t - 90

-36t - 90

Therefore, the simplest form of the given equation [9(-7t + 4t — 10) — 9t] is [-36t - 90].

Know more about the simplest terms here:

https://brainly.com/question/197624

#SPJ1

what is the name of this system? consistent and dependent, consistent and independent, or inconsistent?

Answers

If a system has at least one solution, it is said to be consistent .

For system to be independent,

If a consistent system has exactly one solution, it is independent .

For system to be dependent,

If a consistent system has an infinite number of solutions, it is dependent .

The graph is given in the question tab which has exactly one solution.

So the given system is consistent and independent.

14 Betsy works as waitress. Today she worked an 8 hour shift, and was paid $ 92.00. Plot
a graph of the amount Betsy earns against the time she works for. Then find how much
Betsy is paid per hour. Write an equation using Y = MX + B Form.

Answers

Answer:

11.50 per hour

Step-by-step explanation:

Y = 92

8 = Mx

Y = mx

92 = m(8)

m = 11.50 per hour

w/2+3=5 help w/2 is a fraction Btw solve for w pleasee..!

Answers

Answer:

w = 4

Step-by-step explanation:

[tex]\frac{w}{2}[/tex] + 3 = 5 ( subtract 3 from both sides )

[tex]\frac{w}{2}[/tex] = 2 ( multiply both sides by 2 to clear the fraction )

w = 4

In a video game, Shar has to build a pen shaped like a right triangle for her animals. If she needs 5 feet of fence for the shortest side and 13 feet of fence for the longest side,
how many feet of fencing is needed for the entire animal pen?

Answers

Answer:

The answer is 12.

Step-by-step explanation:

First you need to draw a right triangle. Then the shortest side of the triangle is 5 and the longest is 13 so you would have to do a^2 + b^2 = c^2. So the problem would look like this 5^2 + b^2 = 13^2. 5 x 5= 25 and 13 x 13= 169. Now the problem looks like this 25 + b^2 = 169, since you can't do 25 + b you would have to bring it to the other side. Afterwards the problem should look like this b^2 = 144. Now you would have to square the b and 144 so the answer is 12. Sorry for spelling and grammar mistakes. Hoped this helped!

Answer:

15

Step-by-step explanation

Your bank account has -$21 in it and you deposit $6 per day. How much money is in your account after 4 days?

Answers

So the original amount of money in the bank account is given by a negative number: -$21. If you deposit $6 per day during four days then the total amount of money deposited by the fourth day is:

[tex]4\cdot\text{\$}6=\text{\$}24[/tex]

The total amount of money in the account is given by adding the deposited money to the original amount:

[tex]-\text{\$}21+\text{\$}24=\text{\$}3[/tex]

Then the answer is $3.

An orchard has 40 rows of trees. Each row is a set number
of trees longer than the one preceding it. If the first row
has 24 trees and the last row has 180 trees, find the
following:
a. How many trees are there in the whole orchard?
b. How many trees are there in the 21st row of the orchard?
ar:

Answers

Answer:

a)4080 trees

b)104 trees

Step-by-step explanation:

1st - 24 trees

last (40th) - 180 trees

If differences between any row (except 40th) and next row are equal then

2nd - 24 trees+X

3rd - (2nd+X) or 24 trees + 2X

coefficient of X is number of row minus 1, therefore 40th row has 24 trees + 39X, and it is equal to 180. We can make an equation from this:

24tr+39X=180tr

39X=156tr

X=4tr

By this we can find how many trees are there in the 21st, 21st row has 24 trees+20X=24 trees+80 trees=104 trees

To find how many trees are there in the whole orchard, we add them all:

1st - 24 trees, 2nd - 24 trees + X, 3rd - 24 trees + 2X,

easily, it is just 24 trees x 40 + 39X x 20 = 4080 trees

Answer:

(a) 4,080 trees

(b) 104 trees

Step-by-step explanation:

If the first row has 24 trees and the fortieth row has 180 trees, how many additional trees are in each succeeding row?

[tex](40-1)x=180-24[/tex]

[tex]39x=156[/tex]

[tex]x=4[/tex]

That means that each row has 4 more trees than the preceding row. For example, the second row has 28 trees, the third row has 32 trees, and so on. We can write a formula to tell us how many trees are in any given row:

[tex]n=4(r-1)+24[/tex]

where [tex]r[/tex] is the row number between 1 and 40.

(a) 24 + 28 + 32 + 36 + 40 + 44 + 48 + 52 + 56 + 60 + 64 + 68 + 72 + 76 + 80 + 84 + 88 + 92 + 96 + 100 + 104 + 108 + 112 + 116 + 120 + 124 + 128 + 132 + 136 + 140 + 144 + 148 + 152 + 156 + 160 + 164 + 168 + 172 + 176 + 180 = 4,080

(b) Using our formula above:

[tex]n=4(21-1)+24[/tex]

[tex]n=4(20)+24[/tex]

[tex]n=80+24[/tex]

[tex]n=104[/tex]

help meeeeeeeeeeeeeee pleaseeeeeee

Answers

Answer: 10.8 seconds

Step-by-step explanation:

[tex]-16t^2 +170t+40=10\\\\16t^2 -170t-30=0\\ \\ t=\frac{-(-170) \pm \sqrt{(-170)^2 -4(16)(-30)}}{2(16)}\\\\t \approx 10.8 \text{ } (t > 0)[/tex]

im confused on what goes with what
'

Answers

Answer: b

Step-by-step explanation:

Answer: 5

Step-by-step explanation:

you have to divide it so 7 1/2 ÷ 1 1/2

15/2 × 3/2 then you swap the 3/2 to 2/3 so it will be 15/2 × 2/3 you don't have to cross multiplication so you just multiply across so it would be 15×3=30 and then 2×3=6 so it would be 30/6 but that's not the final answer because it and improper fractions so you divide 30÷6=5 and that's how you get 5

use the values of the vertex and point to write the equation of the graph in standard form

Answers

Explanation

The vertex form of a quadratic function is:

[tex]\begin{gathered} y=a(x-h)^2+k \\ \text{ Where} \\ (h,k)\text{ is the vertex of the parabola } \end{gathered}[/tex]

The standard form of a quadratic function is:

[tex]y=ax^2+bx+c[/tex]

We can do the following steps to solve the exercise.

Step 1: We replace the values of h,k, x, and y into the vertex form of a quadratic equation, and we solve for a.

[tex]\begin{gathered} h=1 \\ k=-5 \\ x=3 \\ y=-1 \end{gathered}[/tex][tex]\begin{gathered} y=a(x-h)^{2}+k \\ -1=a(3-1)^2-5 \\ -1=a(2)^2-5 \\ -1=4a-5 \\ \text{ Add 5 from both sides} \\ -1+5=4a-5+5 \\ 4=4a \\ \text{ Divide by 4 from both sides} \\ \frac{4}{4}=\frac{4a}{4} \\ 1=a \end{gathered}[/tex]

Step 2: We replace the values of a,h, and k into the vertex form of a quadratic equation.

[tex]\begin{gathered} y=a(x-h)^{2}+k \\ y=1(x-1)^2-5 \end{gathered}[/tex]

Step 3: We expand the expression inside the parentheses and combine like terms to convert the function into its standard form.

[tex]\begin{gathered} y=1(x-1)^{2}-5 \\ y=(x-1)^2-5 \\ y=(x-1)(x-1)-5 \\ \text{ Apply the distributive property} \\ y=x(x-1)-1(x-1)-5 \\ y=x*x-x*1-1*x-1*-1-5 \\ y=x^2-x-x+1-5 \\ y=x^2-2x-4 \end{gathered}[/tex]Answer

The equation of the graph in standard form is:

[tex]y=x^{2}-2x-4[/tex]

questions in screenshot (odd numbers only)

Answers

Answer:

1. 250x12= 3000. So she earned $3000 in that 12 weeks, if you include the 500 then it would be 3500 dollars if not then it would be 3000 dollars.

2. Divide 75 and 12 and you get 6.25 if you want to estimate. Then the answer would be, all of the fields will be harvested in just 6 days.

3. Divide 54 and 3 and you get 18. 18 weeks if they frame the sanem number each week.

4. If you divide 8,000 and 450 you get 17.7777777778. If you want to estimate then the it would be 18. So it would take about 18 days to fill up the 8,000 gallons of water.

5. This graph is y=20x

6. Don't really know, if the 0 is 360 then 10 and 300 are pretty easy because 10=30. so 0, 10, 20 is 360, 300,240 likes the graph and I would think you can solve this

WILL GIVE BRAINLIEST IF RIGHT

The following is a list of grades that students received on a recent math exam:

77, 78, 81, 62, 63, 65, 81, 92, 93, 89, 91, 78, 66, 75, 78, 80, 91, 93, 95, 93, 91, 88, 76, 89, 91, 96.

The teacher wants to highlight the lower quartile, median, and upper quartile. Which of the following displays would represent the data in this way?

Bar chart
Box plot
Histogram
Stem-and-leaf plot

Answers

The display that would represent the data in the best way that lower quartile, median, and upper quartile would be highlighted would be box ploy. That is option B.

What is box plot?

Box plot which is also called box and whisker plot is a type of graphical representation that is used to represent the five number summary of a data set that include the following:

Minimum,lower quartile, median,upper quartile andMaximum.

Therefore, among the listed displays, the graphical representation method that can best be used is the box plot.

Learn more about upper quartile here:

https://brainly.com/question/28277179

#SPJ1

Answer:

B - Box Plot

Step-by-step explanation:

I took the test :)

What is a number that is both a square number and a multiple of 9.

Answers

Answer:

3

Step-by-step explanation:

I hope it helps and have a wonderful rest of your day!!!! :)

BRAINIEST is appreciated it would really help me!!!!


Miranda has $50 to buy 6 swim caps for the members of a swim team.
Which expression shows the amount of money she will have left if each swim cap costs c dollars?

Answers

Answer:

50÷6=8 R2

Step-by-step explanation:

                       8  

              6  ⟌50        R2

                     - 0    

                 -------------  

                       50  

                      -48  

                 --------------  

                          2  

So, Miranda will have $8 left to buy.

In the figure below, m<1(15x-5)° and m<3(10x + 35)Find x =and m <2 =

Answers

1) You have the following expressions for angles ∠1 and ∠3

m∠1 = 15x - 5

m∠3 = 10x + 35

In order to find the value of x, you consider that angle ∠1 and ∠3 are congruent, that is, they are equal. Then, you equal the algebraic expressions and solve for x, just as follow:

15x - 5 = 10x + 35 subtract 10x bith sides, add 5 both sides

15x - 10x = 35 + 5 simplify similar terms

5x = 40 divide between 5 both sides

x = 40/5

x = 8

Hence, x = 8, and the values of the angles are:

m∠1 = 15x - 5 = 15(8) - 5 = 115

m∠3 = 10x + 35 = 10(8) + 35 = 115

2) To find the measure of the angle ∠2 you consider that the sum of angles ∠1 and ∠2 is equal to 180°. Thus, you obtain:

m∠1 + m∠2 = 180°

m∠2 = 180 - m∠1 = 180 - 115 = 65

Hence, the measure of angle m∠2 = 65°

I don’t really know how to answer this

Answers

A linear equation for the trend line in this scatter plot is y = 4x/3 - 5/3.

How to determine the equation of this line?

In order to determine a linear equation for the trend line (line of best fit) that models the data points contained in the scatter plot, we would have to determine its slope by using the points that trend line passes through.

Mathematically, the slope of a straight line can be calculated by using this formula;

Slope, m = (Change in y-axis, Δy)/(Change in x-axis, Δx)

Slope, m = (y₂ - y₁)/(x₂ - x₁)

Based on the information provided in the scatter plot, the trend line passes through the following points:

Points (x, y) = (5, 5)Points (x, y) = (8, 9)

Substituting the given points into the formula, we have;

Slope, m = (9 - 5)/(8 - 5)

Slope, m = 4/3

At point (5, 5), a linear equation for the line can be calculated by using the point-slope form:

y - y₁ = m(x - x₁)

Where:

m represents the slope.x and y represent the points.c represent the y-intercept.

Substituting the given points into the formula, we have;

y - y₁ = m(x - x₁)

y - 5 = 4/3(x - 5)

y - 5 = 4x/3 - 20/3

y = 4x/3 - 20/3 + 5

y = 4x/3 - 5/3

Read more on slope here: brainly.com/question/3493733

#SPJ1

A line has a slope of 1 and includes the points (-7, -3) and (c, 4). What is the value of c

Answers

Answer:

  c = 0

Step-by-step explanation:

You want the value of c such that point (c, 4) is on the line with slope 1 through point (-7, -3).

Point-slope equation

The point-slope equation of a line with slope m through point (h, k) is ...

  y -k = m(x -h)

The line with slope 1 through point (-7, -3) has equation ...

  y +3 = 1(x +7)

  y = x +4 . . . . . . . subtract 3 and simplify

(c, 4)

The value of x when y=4 can be found using this equation.

  4 = x +4

  0 = x . . . . . . . subtract 4

The value of c is 0.

I need help graghing

Answers

y = (5/3)^x

For x= 0 , y = 1

For x=1,. y= 5/3

For x=2. y = 25/9= 2 + 7/9

For x= 3. y= (25/9)•5/3 = 10/3 + 35/27

For x= 4. y= (10/3 + 35/27)•5/3= 50/9 + 175/81= 625/81

For x= 5 y = (625/81)•5/3= 3125/243

For x=6. y= 15625/243 = 64 + 73/243

For x=7. y=78125/729 = 107+ 122/729

For x=8. y= 390625/2187= 178+ 1339/2187

NOW Part 2 Drawing it

Write an algebraic expression for each verbal expression.1. x more than 7 2. a number less 35 3. 5 times a number4. one-third of a number5. f divided by 106. the quotation of 45 and r7 three times a number plus 168. 18 decreased by 3 times d9. k squared minus 1110. 20 divided by t to the fifth power

Answers

Part 1

'x more than 7' written as an algebraic expression is:

= 7 + x

Part 2

Let the number be n

Therefore, 'a number less 35' written as an algebraic expression:

=n - 35

Part 3

Let the number be y

'5 times a number' written as an algebraic expression:

=5 x y = 5y

Part 4

[tex]\text{One}-\text{third}=\frac{1}{3}[/tex]

Let the number be x.

[tex]\begin{gathered} \text{One}-\text{third of a number}=\frac{1}{3}\text{ of x} \\ =\frac{1}{3}x \end{gathered}[/tex]

Part 5

[tex]f\text{ divided by 10}=\frac{f}{10}[/tex]

A group of friends wants to go to the amusement park. They have no more than $480 to spend on parking and admission. Parking is $8.75, and tickets cost $15.50 per person, including tax. Which inequality can be used to determine pp, the maximum number of people who can go to the amusement park?

Answers

Using inequality, we know that the maximum number of students that can visit the amusement park is 30.

What is inequality?An inequality in mathematics is a relation that compares two numbers or other mathematical expressions in an unequal way. The majority of the time, size comparisons between two numbers on the number line are made.Make use of the following steps to solve an inequality: Step 1: Remove fractions by multiplying all terms by the fractions' lowest common denominator. Step 2 Combine like terms on both sides of the inequality to simplify. Step 3 Obtain the unknown on one side and the integers on the other by adding or subtracting quantities.

So, inequality represents the maximum number of students who can go to the amusement park:

8.75 + 15.50x ≤ 48015.50x ≤ 480 - 8.7515.50x ≤ 471.25x ≤ 471.25/15.50x ≤ 30.4

Rounding off: x ≤ 30

Therefore, using inequality, we know that the maximum number of students that can visit the amusement park is 30.

Know more about inequality here:

https://brainly.com/question/28964520

#SPJ1

Select the correct answer from each drop-down menu.A seashell has the dimensions shown.5 in10 in.What figure best models the shell, and what is the approximate surface area of the shell?The shell is best modeled by aThe approximate surface area of the shell issquare inches.

Answers

Answer:

Cone

100.6in^2

Step-by-step explanation:

Plato answer.

The approximate surface area of the shell is 75π square inches.

How to find the surface area of some object?

Find the area that its outer surfaces possess. Sum of all those surfaces' area is the surface area of the considered object. Right circular cone is the cone in which the line joining peak of the cone to the center of the base of the circle is perpendicular to the surface of its base.

We are given that;

Diameter= 5in

Height=10in

Now,

Substituting these values in the formula for surface area of a cone, we get:

Surface area of a cone = π(5)(5 + 10)

Simplifying, we get:

Surface area of a cone = 75π in²

Therefore, the surface area will be 75π square inches.

Learn more about surface area here:

https://brainly.com/question/15096475

#SPJ5

Please do 2,3,4,5 and show work on how you got the answers. no graphing needed just the points

Answers

1) Coordinates are (2,-6) and (1,-1)

2) Points are (1,2) and (2,8).

Define coordinates.

You do the reverse to determine a point's coordinates in a coordinate system. Start at the point, then move up or down a vertical line until you reach the x-axis. Your x-coordinate is shown there. To get the y-coordinate, repeat the previous step while adhering to a horizontal line.

A coordinate system in geometry is a method for determining the precise location of points or other geometrical objects on a manifold, such as Euclidean space, using one or more numbers, or coordinates.

Given,

Function:

1) f(x) = -x² - 2x + 2

Let x be 1

y = -(1)² - 2(1) +2

y = -1 -2 + 2

y = -1

Points are (1, -1)

Let x be 2

y = -(2)² - 2(2) + 2

y = -4 -4 + 2

y = -6

Points are (2,-6)

2) f(x) = -2x²

Let x be 1

y = 2(1)²

y = 2

Let x be 2

y = 2(2)²

y = 8

So,

Coordinates are (1,2) and (2,8)

To learn more about coordinates, visit:

https://brainly.com/question/27749090

#SPJ1

can someone please help me find the valu of X?

Answers

The midsegment determines a smaller triangle that is similar to the larger triangle. If two triangles are similar, then their side lengths are at the same ratio, so that:

[tex]\begin{gathered} \frac{5x}{45}=\frac{20}{20+16} \\ \frac{5x}{45}=\frac{20}{36} \end{gathered}[/tex]

From this expression, you can determine the value of x:

-First. multiply both sides by 45:

[tex]\begin{gathered} 45(\frac{5x}{45})=45(\frac{20}{36}) \\ 5x=25 \end{gathered}[/tex]

-Second, divide both sides by 5 to determine the value of x:

[tex]\begin{gathered} \frac{5x}{5}=\frac{25}{5} \\ x=5 \end{gathered}[/tex]

The value of x is equal to 5

URGENT Please helpppp,
I will mark brainliest
Geometry A

Answers

Answer:

Step-by-step explanation:

<DCE≅<BCA

BC≅CE

AC≅CD

BA≅DE

PLEASE HELP ASAP
The following table shows lemonade sales at certain temperatures.
1A - What kind of function does the scatter plot model?
1B - What is the equation that models this function? Round to the nearest tenth.
1C - Using your equation, predict the sales for when it was 98 degrees.
for the other one its: The value of a car depreciates over time. The table shows the value of a car over a period of time.
2A - What kind of function does the scatter plot model?
2B - What is the equation that models this function? Round to the nearest tenth.
2C - Using your equation, predict the value of the car at 9 years.

Answers

Answer:

1a the scatter plot models the heart

Given the point slope form of a line equation write in the slope form equation y+1=-3(x-2)

Answers

Given the line in point slope form:

y + 1 = 3(x - 2)

Let's rewrite the equation in slope intercept form.

Using the point slope form:

y - y1 = m(x - x1)

Thus, we have the following:

x1 = -2

y1 = 1

slope, m = 3

To write in slope intercept form, all you have to do is to isolate the y.

Thus,

y + 1 = 3(x - 2)

Expand the parenthesis:

y + 1 = 3x - 6

Subtract 1 from both sides:

y + 1 - 1 = 3x - 6 - 1

y = 3x - 6 - 1

y = 3x - 7

Therefore, the equation in slope intercept form is:

y = 3x - 7

ANSWER:

y = 3x - 7

Answer:

Step-by-step explanation:

1 Divide both sides by -3−3.

-\frac{y+1}{3}=x-2

3

y+1

=x−2

2 Add 22 to both sides.

-\frac{y+1}{3}+2=x

3

y+1

+2=x

3 Regroup terms.

2-\frac{y+1}{3}=x

2−

3

y+1

=x

4 Switch sides.

x=2-\frac{y+1}{3}

x=2−

3

y+1

Complete each equation below so that it shows equivalent fractions.

Answers

1/4 would be equal to 3/12 because you’re multiplying by 3
4/5 would be 8/10 because you’re multiplying by 2
1/6 can be many, but the answer would be 2/12 because you multiplied by two

Answer:

[tex]\frac{3}{12}, \frac{8}{10},\frac{2}{12}[/tex]  

Step-by-step explanation:

→ Find the multiplier in the first one

12 ÷ 4 = 3

→ Multiply onto top

[tex]\frac{3}{12}[/tex]

→ Find multiplier for second one

10 ÷ 5 = 2

→ Multiply

2 × 4 = 8

Other Questions
What are the possible values for p in the equation below? Check all that apply. Ipl = 12 -12 -6 0 1 6 12 24 JUST NEED HELP ON THE LAST ONE AND A VERIFICATION ON EVERYTHING ELSE !!!!!!!!! Hi! May you please help me complete question 4. I'm kinda confused I do not understand how to do these two questions below. A bus goes 500km east from town A to town B in the morning and comes back halfway in the evening travelling west. What is the distance and displacement of the bus? The revenue equation for a certain brand of toothpaste is y = 2.2x, where x is the number of tubes of toothpaste sold andy is the total income for selling x tubes. The cost equation is y = 0.6x + 2000, where x is the number of tubes of toothpastemanufactured and y is the cost of producing x tubes. The following set of axes shows the graph of the cost and revenueequations. For what x-values will the company make a profit? a factory makes 4.8 kilograms of pumpkin pie filling per minute. how many kilograms of pie filling will the factory make in 10 minutes? Solve the quadratic equation using the quadratic formula or by completing the square. Show exact answers in simplified radical form; no rounded decimals. x^2-6x+1=0 Identify the the coefficients of variable terms of the expression -2x^2+8x What rough outline of a policy would you establish to determine who should be allowed to have administrative rights on a computer system with role-based access control? Remember, policies can refer to specific workstations, employee types, customer types, etc. Defend the major tenets of your policy. .: 5 : 6 : 7= 24 :15 : . :. What is the missing number? Why might an infographic use an overwhelming number of colors or fonts?A: to make the information clearerB: to prove the designer is an expertC: to distort dataD: to help interpret data which of the following statements about expansionary fiscal policy is true? a.) it is used to curb inflation. b.) it tends to increase the unemployment rate. c.) it is when government expenditures exceeds tax revenues. d.) it decreases the purchase of goods and services. Calculate the volume of a triangular pyramid 12cm tall and with a base 12cm long and 10cm wide How do people feel about the writing on the wall What is the general reason for a software licensing?(1 point)Responsesto protect the users computer from malwareto protect the intellectual property of the creatorto collect data about the user with permissionto protect the privacy of the users QUESTION Bgraph the line with the given slope and point 2 stepsthere are two steps with in 1 question Please assist me in knowing how to figure these out. If angle YWV is 48 degrees, what is the measure of angle Y?Required to answer. Single choice. 52429048 which of the following number lines represents the inequality x> 1.5?